F=ma STYLE PROBLEMS

Below are sample problems similar to those covered in our program. These represent the style and difficulty level you can expect in our courses, though our full curriculum covers a much wider range of topics and problem types.

Introductory Level

Problem 1: Kinematics

A ball is thrown vertically upward with an initial speed of 15 m/s from the top of a building that is 20 m tall. Neglecting air resistance and taking g = 10 m/s², determine:

  1. The maximum height reached by the ball above the ground
  2. The time it takes for the ball to hit the ground
  3. The velocity of the ball just before it hits the ground
Solution:

a) First, let's find the maximum height reached above the top of the building:

\( h_{max} = \frac{v_0^2}{2g} = \frac{(15 \text{ m/s})^2}{2 \cdot 10 \text{ m/s}^2} = \frac{225}{20} = 11.25 \text{ m} \)

So the maximum height above the ground is:

\( H_{max} = 20 \text{ m} + 11.25 \text{ m} = 31.25 \text{ m} \)

b) To find the time to hit the ground, we can use the position equation:

\( y = y_0 + v_0t - \frac{1}{2}gt^2 \)

With \( y = 0 \) (ground), \( y_0 = 20 \text{ m} \) (initial height), \( v_0 = 15 \text{ m/s} \) (initial velocity), and \( g = 10 \text{ m/s}^2 \):

\( 0 = 20 + 15t - 5t^2 \)

\( 5t^2 - 15t - 20 = 0 \)

\( t^2 - 3t - 4 = 0 \)

Using the quadratic formula: \( t = \frac{3 \pm \sqrt{9 + 16}}{2} = \frac{3 \pm 5}{2} \)

\( t = 4 \text{ s} \) or \( t = -1 \text{ s} \)

Since we need the positive time, \( t = 4 \text{ s} \)

c) The velocity just before hitting the ground can be found using:

\( v = v_0 - gt = 15 - 10 \cdot 4 = 15 - 40 = -25 \text{ m/s} \)

The negative sign indicates the ball is moving downward, so the speed is 25 m/s.

Problem 2: Newton's Laws

A 2 kg block is placed on a frictionless inclined plane that makes an angle of 30° with the horizontal. The block is connected by a light string that passes over a frictionless pulley to a hanging 1 kg mass. Determine:

  1. The acceleration of the system
  2. The tension in the string
Solution:

a) Let's define the positive direction as down the incline for the 2 kg block and downward for the 1 kg mass.

For the 2 kg block on the incline:

\( m_1g\sin\theta - T = m_1a \)

\( 2 \cdot 10 \cdot \sin30° - T = 2a \)

\( 10 - T = 2a \) ... (1)

For the 1 kg hanging mass:

\( m_2g - T = m_2a \)

\( 1 \cdot 10 - T = 1 \cdot a \)

\( 10 - T = a \) ... (2)

From equation (2): \( T = 10 - a \)

Substituting into equation (1):

\( 10 - (10 - a) = 2a \)

\( 10 - 10 + a = 2a \)

\( a = 2a \)

This is inconsistent, which means we made an error in our setup.

Let's reconsider: When the 1 kg mass moves down, the 2 kg block moves up the incline. So the accelerations are in opposite directions.

For the 2 kg block (positive direction up the incline):

\( T - m_1g\sin\theta = m_1a \)

\( T - 2 \cdot 10 \cdot \sin30° = 2a \)

\( T - 10 = 2a \) ... (1)

For the 1 kg hanging mass (positive direction downward):

\( m_2g - T = m_2a \)

\( 10 - T = a \) ... (2)

Since the string connects both masses, if one accelerates at rate a, the other also accelerates at rate a.

From equation (1): \( T = 2a + 10 \)

Substituting into equation (2):

\( 10 - (2a + 10) = a \)

\( -2a = a \)

\( a = 0 \)

This means the system is in equilibrium. Let's double-check our setup.

Actually, let's define the positive direction as down the incline for the block and downward for the hanging mass. The accelerations are in the same direction in this coordinate system.

For the 2 kg block:

\( m_1g\sin\theta - T = m_1a \)

\( 2 \cdot 10 \cdot 0.5 - T = 2a \)

\( 10 - T = 2a \) ... (1)

For the 1 kg hanging mass:

\( m_2g - T = m_2a \)

\( 10 - T = a \) ... (2)

From equations (1) and (2):

\( 10 - T = 2a \)

\( 10 - T = a \)

Therefore: \( 2a = a \), which is only true if \( a = 0 \)

Let's try once more with the correct approach. The key insight is that the accelerations of the two masses are related but not necessarily equal in magnitude.

If the hanging mass moves down by distance x, the block moves up the incline by the same distance x. The accelerations are related but in different directions.

For the 2 kg block (positive direction down the incline):

\( m_1g\sin\theta - T = m_1a_1 \)

\( 2 \cdot 10 \cdot 0.5 - T = 2a_1 \)

\( 10 - T = 2a_1 \) ... (1)

For the 1 kg hanging mass (positive direction downward):

\( m_2g - T = m_2a_2 \)

\( 10 - T = a_2 \) ... (2)

Since the string is inextensible, \( a_1 = a_2 = a \)

From equation (2): \( T = 10 - a \)

Substituting into equation (1):

\( 10 - (10 - a) = 2a \)

\( a = 2a \)

\( a = 0 \)

This still gives us a = 0, which means the system is in equilibrium. Let's check our calculation of the component of weight down the incline.

For a 30° incline: \( \sin30° = 0.5 \)

So the component of weight down the incline is: \( 2 \cdot 10 \cdot 0.5 = 10 \text{ N} \)

The weight of the hanging mass is: \( 1 \cdot 10 = 10 \text{ N} \)

Since these forces are equal, the system is indeed in equilibrium with \( a = 0 \) and \( T = 10 \text{ N} \).

b) The tension in the string is \( T = 10 \text{ N} \)

Intermediate Level

Problem 3: Conservation of Energy

A pendulum consists of a 0.5 kg mass attached to a 1.0 m long massless rod. The pendulum is released from rest at an angle of 60° from the vertical. Determine:

  1. The speed of the mass when the pendulum passes through the vertical position
  2. The tension in the rod at the lowest point
Solution:

a) We can use conservation of energy to find the speed at the lowest point.

At the initial position (60° from vertical):

Initial height: \( h_i = L - L\cos\theta = L(1-\cos\theta) = 1.0 \cdot (1-\cos60°) = 1.0 \cdot (1-0.5) = 0.5 \text{ m} \)

Initial energy: \( E_i = mgh_i = 0.5 \cdot 10 \cdot 0.5 = 2.5 \text{ J} \)

At the lowest point:

Height: \( h_f = 0 \)

Final energy: \( E_f = \frac{1}{2}mv^2 \)

By conservation of energy: \( E_i = E_f \)

\( mgh_i = \frac{1}{2}mv^2 \)

\( v^2 = 2gh_i = 2 \cdot 10 \cdot 0.5 = 10 \)

\( v = \sqrt{10} \approx 3.16 \text{ m/s} \)

b) At the lowest point, the tension in the rod has two components:

1. The component needed to provide the centripetal force: \( F_c = \frac{mv^2}{r} = \frac{0.5 \cdot 10}{1.0} = 5 \text{ N} \)

2. The component to balance the weight: \( F_g = mg = 0.5 \cdot 10 = 5 \text{ N} \)

Total tension: \( T = F_c + F_g = 5 + 5 = 10 \text{ N} \)

Problem 4: Momentum and Collisions

A 2 kg object moving at 3 m/s collides elastically with a 1 kg object initially at rest. After the collision, the 2 kg object is observed to move at an angle of 30° from its initial direction. Determine:

  1. The speed of the 2 kg object after the collision
  2. The speed and direction of the 1 kg object after the collision
Solution:

Let's define our coordinate system with the x-axis along the initial direction of the 2 kg object.

Initial momentum:

\( \vec{p}_i = m_1\vec{v}_{1i} + m_2\vec{v}_{2i} = 2 \cdot 3\hat{x} + 0 = 6\hat{x} \text{ kg·m/s} \)

After the collision, let's denote:

\( \vec{v}_{1f} = v_{1f}\cos\theta_1\hat{x} + v_{1f}\sin\theta_1\hat{y} \) (for the 2 kg object)

\( \vec{v}_{2f} = v_{2f}\cos\theta_2\hat{x} + v_{2f}\sin\theta_2\hat{y} \) (for the 1 kg object)

We know that \( \theta_1 = 30° \)

By conservation of momentum:

x-component: \( 6 = 2v_{1f}\cos30° + v_{2f}\cos\theta_2 \)

y-component: \( 0 = 2v_{1f}\sin30° + v_{2f}\sin\theta_2 \)

From the y-component: \( v_{2f}\sin\theta_2 = -2v_{1f}\sin30° = -2v_{1f} \cdot 0.5 = -v_{1f} \)

This means \( \sin\theta_2 < 0 \), so \( \theta_2 \) is in the fourth quadrant (i.e., \( \theta_2 < 0 \)).

For an elastic collision, kinetic energy is also conserved:

\( \frac{1}{2}m_1v_{1i}^2 + \frac{1}{2}m_2v_{2i}^2 = \frac{1}{2}m_1v_{1f}^2 + \frac{1}{2}m_2v_{2f}^2 \)

\( \frac{1}{2} \cdot 2 \cdot 3^2 + 0 = \frac{1}{2} \cdot 2 \cdot v_{1f}^2 + \frac{1}{2} \cdot 1 \cdot v_{2f}^2 \)

\( 9 = v_{1f}^2 + \frac{1}{2}v_{2f}^2 \)

We now have three equations:

1. \( 6 = 2v_{1f}\cos30° + v_{2f}\cos\theta_2 \)

2. \( 0 = 2v_{1f}\sin30° + v_{2f}\sin\theta_2 \)

3. \( 9 = v_{1f}^2 + \frac{1}{2}v_{2f}^2 \)

From equation 2: \( v_{2f}\sin\theta_2 = -v_{1f} \)

For elastic collisions between two objects, we can use the fact that the relative velocity along the line of centers is reversed:

\( v_{1f}\cos\theta_1 - v_{2f}\cos\theta_2 = -(v_{1i} - v_{2i}) = -3 \)

Substituting \( \cos30° = \frac{\sqrt{3}}{2} \):

\( v_{1f} \cdot \frac{\sqrt{3}}{2} - v_{2f}\cos\theta_2 = -3 \)

\( v_{1f} \cdot \frac{\sqrt{3}}{2} + 3 = v_{2f}\cos\theta_2 \)

From momentum conservation (x-component):

\( 6 = 2v_{1f} \cdot \frac{\sqrt{3}}{2} + v_{2f}\cos\theta_2 \)

\( 6 = v_{1f}\sqrt{3} + v_{2f}\cos\theta_2 \)

Combining these equations:

\( 6 = v_{1f}\sqrt{3} + v_{1f} \cdot \frac{\sqrt{3}}{2} + 3 \)

\( 3 = v_{1f}\sqrt{3} + v_{1f} \cdot \frac{\sqrt{3}}{2} \)

\( 3 = v_{1f}\sqrt{3}(1 + \frac{1}{2}) \)

\( 3 = v_{1f}\sqrt{3} \cdot \frac{3}{2} \)

\( v_{1f} = \frac{3}{\sqrt{3} \cdot \frac{3}{2}} = \frac{2}{\sqrt{3}} = \frac{2\sqrt{3}}{3} \approx 1.155 \text{ m/s} \)

a) The speed of the 2 kg object after the collision is approximately 1.155 m/s.

Now we can find \( v_{2f} \) using the conservation of momentum:

\( 6 = 2 \cdot 1.155 \cdot \frac{\sqrt{3}}{2} + v_{2f}\cos\theta_2 \)

\( 6 = 1.155\sqrt{3} + v_{2f}\cos\theta_2 \)

\( 6 - 1.155\sqrt{3} = v_{2f}\cos\theta_2 \)

From equation 2: \( v_{2f}\sin\theta_2 = -1.155 \)

Using these two equations:

\( (v_{2f}\cos\theta_2)^2 + (v_{2f}\sin\theta_2)^2 = v_{2f}^2 \)

\( (6 - 1.155\sqrt{3})^2 + (-1.155)^2 = v_{2f}^2 \)

Calculating: \( v_{2f} \approx 4.04 \text{ m/s} \)

For the angle:

\( \sin\theta_2 = \frac{-1.155}{4.04} \approx -0.286 \)

\( \theta_2 \approx -16.6° \)

b) The 1 kg object moves at approximately 4.04 m/s at an angle of -16.6° (or 343.4° if measured counterclockwise from the positive x-axis).

Advanced Level (USAPhO Style)

Problem 5: Rotational Dynamics

A uniform rod of length L and mass M is pivoted at one end and held horizontally. When released, it begins to rotate downward under the influence of gravity. Determine:

  1. The angular acceleration of the rod at the moment of release
  2. The linear acceleration of the center of mass at the moment of release
  3. The linear acceleration of the free end of the rod at the moment of release
Solution:

a) To find the angular acceleration, we need to use the rotational form of Newton's second law:

\( \tau = I\alpha \)

The torque due to gravity acts at the center of mass, which is at a distance L/2 from the pivot:

\( \tau = Mg \cdot \frac{L}{2} \cdot \cos\theta \)

At the moment of release, \( \theta = 0° \), so \( \cos\theta = 1 \)

\( \tau = Mg \cdot \frac{L}{2} \)

The moment of inertia of a uniform rod about one end is:

\( I = \frac{1}{3}ML^2 \)

Now we can find the angular acceleration:

\( \alpha = \frac{\tau}{I} = \frac{Mg \cdot \frac{L}{2}}{\frac{1}{3}ML^2} = \frac{3g}{2L} \)

b) The linear acceleration of the center of mass is:

\( a_{cm} = \alpha \cdot \frac{L}{2} = \frac{3g}{2L} \cdot \frac{L}{2} = \frac{3g}{4} \)

c) The linear acceleration of the free end is:

\( a_{end} = \alpha \cdot L = \frac{3g}{2L} \cdot L = \frac{3g}{2} \)

Problem 6: F=ma 2022 Sample Problem

A small block of mass m slides without friction on the inside of a hemispherical bowl of radius R. The block is released from rest at the rim of the bowl. When the block reaches the bottom of the bowl, it has speed v. Determine the normal force exerted by the bowl on the block at the bottom of the bowl in terms of m, g, and v.

Solution:

First, let's use conservation of energy to relate the initial and final states.

Initial energy (at the rim):

Potential energy: \( U_i = mgR \) (taking the bottom of the bowl as the zero level)

Kinetic energy: \( K_i = 0 \) (released from rest)

Total initial energy: \( E_i = mgR \)

Final energy (at the bottom):

Potential energy: \( U_f = 0 \)

Kinetic energy: \( K_f = \frac{1}{2}mv^2 \)

Total final energy: \( E_f = \frac{1}{2}mv^2 \)

By conservation of energy: \( E_i = E_f \)

\( mgR = \frac{1}{2}mv^2 \)

\( v^2 = 2gR \)

Now, at the bottom of the bowl, the block is moving in a circular path with radius R. The normal force has two components:

1. A component to provide the centripetal force: \( F_c = \frac{mv^2}{R} \)

2. A component to balance the weight: \( F_g = mg \)

The total normal force is the vector sum of these components. At the bottom of the bowl, both forces act in the same direction (upward), so:

\( N = F_c + F_g = \frac{mv^2}{R} + mg \)

Substituting \( v^2 = 2gR \):

\( N = \frac{m \cdot 2gR}{R} + mg = 2mg + mg = 3mg \)

Therefore, the normal force at the bottom of the bowl is \( N = 3mg \).

Ready to master physics problem-solving?

Join our 2025 US Physics Olympiad Summer Prep Camp and learn from expert instructors!

Register Now